14
$\begingroup$

Let us regard the $n\times n$ matrices as operators on the $n$-dimensional $\ell_p$ space; that is, we consider them as linear operators $\ell_p^n\to \ell_p^n$. When $p=2$, $M_n$ is a C*-algebra and we have

$0 \leqslant A \leqslant B \implies \|A\|\leqslant \|B\|$.

Here $\|A\|$ denotes the operator norm of a map $A\colon \ell_2^n\to \ell_2^n$. What about other $p\in [1,\infty]$?

Fix $p\in [1,\infty]$. Is it true that there exists $K>0$ such that for every $n$ and for all $A,B\colon \ell_p^n\to \ell_p^n$ with $0\leqslant A\leqslant B$ (meaning that $A$ and $B$ are self-adjoint and non-negative semi-definite) we have $$\|A\|_{\ell_p^n\to\ell_p^n}\leqslant K\|B\|_{\ell_p^n\to\ell_p^n}?$$

My feeling is that it should be true for $p\in (1,\infty)$. For $p=1$ or $p=\infty$ there is an easy counter-example with $K=1$. Take

$$A=\left[\begin{smallmatrix}2&1\\1& \tfrac{1}{2}\end{smallmatrix}\right],\;\;B = \left[\begin{smallmatrix}\tfrac{5}{2}&0\\0& \tfrac{5}{2}\end{smallmatrix}\right]. $$

Then $0\leqslant A\leqslant B$ yet for $p\in \{1,\infty\}$ we have $\|A\|_{\ell_p^2\to\ell_p^2} = 3$ whereas $\|B\|_{\ell_p^2\to\ell_p^2}=\tfrac{5}{2}$.

In the language of this thread: Monotone matrix norms, I ask whether the operator $\ell_p$-norms are monotone, that is, if we can take $K=1$. user147215 cleverly shows that this is not the case when $p\neq 2$.

Possible approach: It is not inconceivable that using some Riesz–Thorin-type argument we could show that the operator $\ell_p$-norms are indeed monotone for $p$ is some neighbourhood of 2.

$\endgroup$
3
  • $\begingroup$ Please share the counterexample for $p=1,\infty$, if you don't mind. $\endgroup$
    – user147194
    May 1, 2014 at 18:38
  • $\begingroup$ Since you ask if there is a $K=K_p$ independent of $n$; my instinct (which I haven't checked) is to find some $n$ and some $0\leq A \leq I_n$ with $\Vert A\Vert_{p\to p} > 1$ for some $p\neq 2$, and then look at tensor powers of $A$. $\endgroup$
    – Yemon Choi
    May 1, 2014 at 23:07
  • $\begingroup$ The answer is no, Tomek. Use a Kashin decomposition of $L_p^n$ to see that a random orthogonal projection has norm at least $Cn^{|1/p-1/2|}$. $\endgroup$ May 2, 2014 at 1:15

2 Answers 2

4
$\begingroup$

The answer is no, Tomek. Use a Kashin decomposition of $L_p^n$, $1\le p < 2$, to see that there are orthogonal projections whose norms as operators on $L_p^n$ are of order $Cn^{|1/p-1/2|}$. (Kashin proved that for $1\le p < 2$ there is an orthogonal decomposition $A+B$ of $n$-space s.t. if $x \in A \cup B$, then $\|x\|_p \le \|x\|_2 \le C\|x\|_p$, where I am using the uniform probability measure on $\{1,\dots,n\}$ rather than counting measure to define the norms.)

The case $p>2$ follows by duality.

B. S. Kashin, Sections of some finite-dimensional bodies and classes of smooth functions. Izv. Acad. Nauk SSSR41 (1997), 334--351.

$\endgroup$
4
  • $\begingroup$ Thank you. I didn't know Kashin's decomposition. Actually this suggests a question of whether the converse holds: if this condition is satisfied, must the induced operator norms come from Euclidean spaces? I think I can prove that the answer is no if the finite-dimensional spaces $E_n$ (which now generalise $\ell_p^n$ in the statement) are ranges of initial projections of a symmetric basis in a space which contains $\ell_1^n$ or $\ell_\infty^n$ uniformly complemented. $\endgroup$ May 2, 2014 at 13:00
  • 1
    $\begingroup$ Sounds likely. The condition is equivalent to saying that there is a constant $C$ s.t. every orthogonal basis has unconditional constant at most $C$. I bet that this implies that the norm is $f(C)$ equivalent to the Euclidean norm. $\endgroup$ May 2, 2014 at 13:10
  • $\begingroup$ I'm glad to hear that; looks like a nice exercise! $\endgroup$ May 2, 2014 at 13:14
  • $\begingroup$ Let me point out that for $K=1$ and any $n$-dimensional space the claim follows from the Kakutani-Sobczyk-Bohnenblust theorem characterising isometrically Hilbert spaces as those Banach spaces whose all 2-dimensional subspaces are 1-complemented. $\endgroup$ Jun 5, 2014 at 23:12
2
$\begingroup$

The original version of the question, which asked for $\|A\|_{\ell_p^n\to\ell_p^n}\leqslant K\|B\|_{\ell_p^n\to\ell_p^n}$ with $K = K(p)$, was more interesting (and I don't have an answer to that one yet).

But if you insist on $K=1$, then the inequality fails for all $p\ne 2$. Let $n=3$, $$A=\begin{pmatrix} 2/3 & -1/3 & -1/3 \\ -1/3 & 2/3 & -1/3 \\ -1/3 & -1/3 & 2/3 \end{pmatrix},\qquad B = \begin{pmatrix} 1 & 0 & 0 \\ 0 & 1 & 0 \\ 0 & 0 & 1 \end{pmatrix}$$ That is, $B=I$ and $A=I-P$ where $P$ is the orthogonal projection onto the line $x_1=x_2=x_3$. What $A$ does is subtract the mean of $x_i$ from each coordinate.

Subtracting the mean does not increase the $\ell_2$ norm of a vector, but it may increase $\ell_p$ for any other $p$. I will show that $\|A\|_{\ell_p^n\to\ell_p^n}>1= \|B\|_{\ell_p^n\to\ell_p^n}$.

Fix $p\in (1,\infty)\setminus \{2\}$ and let $x = (2^{1/(p-1)},-1,-1)^T$. For $p\in (1,\infty)$ the function $$\phi(t) = \sum_i |x_i-t|^p$$ is strictly convex, and attains its minimum when $\phi'(t)=0$, namely at the point with $$ \sum_i |x_i-t|^{p-1}\operatorname{sign}(x_i-t)=0 \tag{1}$$ For $x$ as above, (1) is satisfied with $t=0$. Therefore, $\phi(t)>\|x\|_p^p$ for all $t\ne 0$. Applying $A$ amounts to replacing each $x_i$ with $x_i-t$, where $t= (2^{1/(p-1)}-2)/3$. When $p\ne 2$, we have $t \ne 0$, hence $\|Ax\|_p> \|x\|_p$.

$\endgroup$
0

Your Answer

By clicking “Post Your Answer”, you agree to our terms of service and acknowledge you have read our privacy policy.

Not the answer you're looking for? Browse other questions tagged or ask your own question.